From 4e9676fc85bf0ba0ae326a4bbdcc76d9d387bb8d Mon Sep 17 00:00:00 2001 From: "W. Trevor King" Date: Fri, 20 Apr 2012 17:48:23 -0400 Subject: [PATCH] Add solution text for Serway and Jewett v8's 25.7, .12, and .16. --- .../Serway_and_Jewett_8/problem25.07.tex | 12 +++++++++ .../Serway_and_Jewett_8/problem25.12.tex | 18 +++++++++++++ .../Serway_and_Jewett_8/problem25.16.tex | 26 +++++++++++++++++++ 3 files changed, 56 insertions(+) diff --git a/latex/problems/Serway_and_Jewett_8/problem25.07.tex b/latex/problems/Serway_and_Jewett_8/problem25.07.tex index 238876f..315852d 100644 --- a/latex/problems/Serway_and_Jewett_8/problem25.07.tex +++ b/latex/problems/Serway_and_Jewett_8/problem25.07.tex @@ -35,4 +35,16 @@ label("Top view", (u/2, 0), align=S); \end{problem*} \begin{solution} +This is just like a gravitational pendulum, except that the external +force is $qE$ from the electric field instead of the usual $mg$ from +the gravitational field. We can solve the problem by conserving +energy between the two snapshots shown in the figure. The +gravitational potential energy $mgh$ is now $qEh$, so +\begin{align} + qEL(1-\cos(\theta)) &= \frac{1}{2} mv^2 \\ + v &= \sqrt{\frac{2qEL(1-\cos(\theta))}{m}} + = \sqrt{\frac{2\cdot(2.00\E{-6}\U{C})\cdot(300\U{V/m}) + \cdot(1.50\U{m})\cdot(1-\cos(60.0\dg))}{0.0100\U{kg}}} + = \ans{0.300\U{m/s}} \;. +\end{align} \end{solution} diff --git a/latex/problems/Serway_and_Jewett_8/problem25.12.tex b/latex/problems/Serway_and_Jewett_8/problem25.12.tex index b3a8dc2..9eda325 100644 --- a/latex/problems/Serway_and_Jewett_8/problem25.12.tex +++ b/latex/problems/Serway_and_Jewett_8/problem25.12.tex @@ -23,4 +23,22 @@ label("$d$", (u/2, 0), align=S); \end{problem*} \begin{solution} +\Part{a} +Using the formula for electric potential due to a point charge, we have +\begin{equation} + V_A = k\frac{-15.0\U{nC}}{d} + k\frac{27.0\U{nC}}{d} + = \frac{8.99\E{-9}\U{Nm$^2$/C$^2$}}{2.00\E{-2}\U{m}} + \cdot(-15.0+27.0)\E{-9}\U{C} + = \ans{5.39\E{3}\U{V}} = \ans{5.39\U{kV}} \;. +\end{equation} +Because electric potential is a scalar quantity, there's no need to +mess around with any pesky vectors. + +\Part{b} +The same formula holds, but now there is half the distance between the +charges and the point of interest. +\begin{equation} + V_B = \frac{k}{d/2}\cdot(-15.0+27.0)\E{-9}\U{C} = 2V_A + = \ans{10.8\U{kV}} \;. +\end{equation} \end{solution} diff --git a/latex/problems/Serway_and_Jewett_8/problem25.16.tex b/latex/problems/Serway_and_Jewett_8/problem25.16.tex index bdf7ea8..f6e2fa7 100644 --- a/latex/problems/Serway_and_Jewett_8/problem25.16.tex +++ b/latex/problems/Serway_and_Jewett_8/problem25.16.tex @@ -22,4 +22,30 @@ label("$d$", (u/2, 0), align=S); \end{problem*} \begin{solution} +\Part{a} +Using the formula for electric potential due to a point charge, we have +\begin{equation} + V_A = k\frac{Q}{d} + k\frac{2Q}{d\sqrt{2}} + = k\frac{Q}{d}\p({1+\sqrt{2}}) + = \ans{5.43\E{3}\U{V}} = \ans{5.43\U{kV}} \;, +\end{equation} +where we used the pythagorean theorem to find the distance from the +$2Q$ charge to $A$ ($\sqrt{d^2 + d^2} = \sqrt{2d^2} = d\sqrt{2}$). + +\Part{b} +The geometry is flipped for $B$, so we have +\begin{equation} + V_A = k\frac{Q}{d\sqrt{2}} + k\frac{2Q}{d} + = k\frac{Q}{d}\p({2+\frac{1}{\sqrt{2}}}) + = \ans{6.08\E{-15}\U{V}} = \ans{6.08\U{kV}} \;, +\end{equation} + +\Part{c} +\begin{equation} + \Delta V_{AB} = V_B - V_A + = k\frac{Q}{d}\p({2+\frac{1}{\sqrt{2}}}) - k\frac{Q}{d}\p({1+\sqrt{2}}) + = k\frac{Q}{d}\p({1 + \frac{1}{\sqrt{2}} - \frac{2}{\sqrt{2}}}) + = k\frac{Q}{d}\p({1 - \frac{1}{\sqrt{2}}}) + = \ans{658\U{V}} \;. +\end{equation} \end{solution} -- 2.26.2